aboutsummaryrefslogtreecommitdiff
path: root/posts/2019-03-13-a-tail-of-two-densities.md
diff options
context:
space:
mode:
Diffstat (limited to 'posts/2019-03-13-a-tail-of-two-densities.md')
-rw-r--r--posts/2019-03-13-a-tail-of-two-densities.md1290
1 files changed, 0 insertions, 1290 deletions
diff --git a/posts/2019-03-13-a-tail-of-two-densities.md b/posts/2019-03-13-a-tail-of-two-densities.md
deleted file mode 100644
index 44a1d50..0000000
--- a/posts/2019-03-13-a-tail-of-two-densities.md
+++ /dev/null
@@ -1,1290 +0,0 @@
----
-date: 2019-03-13
-title: A Tail of Two Densities
-template: post
-comments: true
----
-
-This is Part 1 of a two-part post where I give an introduction to
-the mathematics of differential privacy.
-
-Practically speaking, [differential privacy](https://en.wikipedia.org/wiki/Differential_privacy)
-is a technique of perturbing database queries so that query results do not
-leak too much information while still being relatively accurate.
-
-This post however focuses on the mathematical aspects of differential privacy, which is
-a study of [tail bounds](https://en.wikipedia.org/wiki/Concentration_inequality)
-of the divergence between
-two probability measures, with the end goal of applying it to [stochastic
-gradient descent](https://en.wikipedia.org/wiki/Stochastic_gradient_descent).
-This post should be suitable for anyone familiar with probability theory.
-
-I start with the definition of $\epsilon$-differential privacy
-(corresponding to max divergence), followed by
-$(\epsilon, \delta)$-differential privacy (a.k.a. approximate
-differential privacy, corresponding to the $\delta$-approximate max
-divergence). I show a characterisation of the $(\epsilon, \delta)$-differential privacy
-as conditioned $\epsilon$-differential privacy.
-Also, as examples, I illustrate the $\epsilon$-dp with Laplace mechanism and, using
-some common tail bounds, the approximate dp with the Gaussian mechanism.
-
-Then I continue to show the effect of combinatorial
-and sequential compositions of randomised queries (called mechanisms)
-on privacy by stating and proving the composition theorems for differential privacy,
-as well as the effect of mixing mechanisms, by presenting the subsampling theorem
-(a.k.a. amplification theorem).
-
-In [Part 2](/posts/2019-03-14-great-but-manageable-expectations.html), I discuss the Rényi differential privacy, corresponding to
-the Rényi divergence, a study of the [moment generating functions](https://en.wikipedia.org/wiki/Moment-generating_function) of the
-divergence between probability measures to derive the tail bounds.
-
-Like in Part 1, I prove a composition theorem and a subsampling theorem.
-
-I also attempt to reproduce a seemingly better moment bound for the
-Gaussian mechanism with subsampling, with one intermediate step which I
-am not able to prove.
-
-After that I explain the Tensorflow implementation of differential privacy
-in its [Privacy](https://github.com/tensorflow/privacy/tree/master/privacy) module,
-which focuses on the differentially private stochastic gradient descent
-algorithm (DP-SGD).
-
-Finally I use the results from both Part 1 and Part 2 to obtain some privacy
-guarantees for composed subsampling queries in general, and for DP-SGD in particular.
-I also compare these privacy guarantees.
-
-**Acknowledgement**. I would like to thank
-[Stockholm AI](http://stockholm.ai) for introducing me to the subject
-of differential privacy. Thanks to Amir Hossein Rahnama for hosting the discussions at
-Stockholm AI.
-Thanks to (in chronological order) Reynaldo
-Boulogne, Martin Abedi, Ilya Mironov, Kurt Johansson, Mark Bun, Salil
-Vadhan, Jonathan Ullman, Yuanyuan Xu and Yiting Li for communication and
-discussions. Also thanks to the [r/MachineLearning](https://www.reddit.com/r/MachineLearning/)
-community for comments and suggestions which result in improvement of
-readability of this post. The research was done while working at [KTH Department of
-Mathematics](https://www.kth.se/en/sci/institutioner/math).
-
-*If you are confused by any notations, ask me or try [this](/notations.html).
-This post (including both Part 1 and Part2) is licensed under
-[CC BY-SA](https://creativecommons.org/licenses/by-sa/4.0/)
-and [GNU FDL](https://www.gnu.org/licenses/fdl.html).*
-
-The gist of differential privacy
---------------------------------
-
-If you only have one minute, here is what differential privacy is about:
-
-Let $p$ and $q$ be two probability densities, we define the *divergence
-variable*[^dv] of $(p, q)$ to be
-
-$$L(p || q) := \log {p(\xi) \over q(\xi)}$$
-
-where $\xi$ is a random variable distributed according to $p$.
-
-Roughly speaking, differential privacy is the study of the tail bound of
-$L(p || q)$: for certain $p$s and $q$s, and for
-$\epsilon > 0$, find $\delta(\epsilon)$ such that
-
-$$\mathbb P(L(p || q) > \epsilon) < \delta(\epsilon),$$
-
-where $p$ and $q$ are the laws of the outputs of a randomised functions
-on two very similar inputs.
-Moreover, to make matters even simpler, only three situations need to be considered:
-
-1. (General case) $q$ is in the form of $q(y) = p(y + \Delta)$ for some bounded constant $\Delta$.
-2. (Compositions) $p$ and $q$ are combinatorial or sequential compositions of some simpler $p_i$'s and $q_i$'s respectively
-3. (Subsampling) $p$ and $q$ are mixtures / averages of some simpler $p_i$'s and $q_i$'s respectively
-
-In applications, the inputs are databases and the randomised functions
-are queries with an added noise, and the tail bounds give privacy
-guarantees. When it comes to gradient descent, the input is the training
-dataset, and the query updates the parameters, and privacy is achieved
-by adding noise to the gradients.
-
-Now if you have an hour\...
-
-[^dv]: For those who have read about differential privacy and never heard
-of the term \"divergence variable\", it is closely related to the notion of \"privacy loss\",
-see the paragraph under Claim 6 in [Back to approximate differential privacy](#back-to-approximate-differential-privacy).
-I defined the term this way so that we can focus on the more general stuff:
-compared to the privacy loss $L(M(x) || M(x'))$, the term $L(p || q)$ removes
-the \"distracting information\" that $p$ and $q$ are related to databases,
-queries, mechanisms etc., but merely probability laws. By removing the distraction,
-we simplify the analysis. And once we are done with the analysis of $L(p || q)$,
-we can apply the results obtained in the general setting to the special case
-where $p$ is the law of $M(x)$ and $q$ is the law of $M(x')$.
-
-$\epsilon$-dp
--------------
-
-**Definition (Mechanisms)**. Let $X$ be a
-space with a metric $d: X \times X \to \mathbb N$. A *mechanism* $M$ is
-a function that takes $x \in X$ as input and outputs a random variable
-on $Y$.
-
-In this post, $X = Z^m$ is the space of datasets of $m$ rows for some
-integer $m$, where each item resides in some space $Z$. In this case the distance
-$d(x, x') := \#\{i: x_i \neq x'_i\}$ is the number of rows that differ
-between $x$ and $x'$.
-
-Normally we have a query $f: X \to Y$, and construct the mechanism $M$
-from $f$ by adding a noise:
-
-$$M(x) := f(x) + \text{noise}.$$
-
-Later, we will also consider mechanisms constructed from composition or mixture of
-other mechanisms.
-
-In this post $Y = \mathbb R^d$ for some $d$.
-
-**Definition (Sensitivity)**. Let
-$f: X \to \mathbb R^d$ be a function. The *sensitivity* $S_f$ of $f$ is
-defined as
-
-$$S_f := \sup_{x, x' \in X: d(x, x') = 1} \|f(x) - f(x')\|_2,$$
-
-where $\|y\|_2 = \sqrt{y_1^2 + ... + y_d^2}$ is the $\ell^2$-norm.
-
-**Definition (Differential
-Privacy)**. A mechanism $M$ is called $\epsilon$*-differential privacy*
-($\epsilon$-dp) if it satisfies the following condition: for all
-$x, x' \in X$ with $d(x, x') = 1$, and for all measureable set
-$S \subset \mathbb R^n$,
-
-$$\mathbb P(M(x) \in S) \le e^\epsilon P(M(x') \in S). \qquad (1)$$
-
-Practically speaking, this means given the results from perturbed query on
-two known databases that differs by one row, it is hard to determine
-which result is from which database.
-
-An example of $\epsilon$-dp mechanism is the Laplace mechanism.
-
-**Definition**. The *Laplace distribution* over $\mathbb R$
-with parameter $b > 0$ has probability density function
-
-$$f_{\text{Lap}(b)}(x) = {1 \over 2 b} e^{- {|x| \over b}}.$$
-
-**Definition**. Let $d = 1$. The *Laplace mechanism* is
-defined by
-
-$$M(x) = f(x) + \text{Lap}(b).$$
-
-**Claim**. The Laplace mechanism with
-
-$$b \ge \epsilon^{-1} S_f \qquad (1.5)$$
-
-is $\epsilon$-dp.
-
-**Proof**. Quite straightforward. Let $p$ and $q$ be the laws
-of $M(x)$ and $M(x')$ respectively.
-
-$${p (y) \over q (y)} = {f_{\text{Lap}(b)} (y - f(x)) \over f_{\text{Lap}(b)} (y - f(x'))} = \exp(b^{-1} (|y - f(x')| - |y - f(x)|))$$
-
-Using triangular inequality $|A| - |B| \le |A - B|$ on the right hand
-side, we have
-
-$${p (y) \over q (y)} \le \exp(b^{-1} (|f(x) - f(x')|)) \le \exp(\epsilon)$$
-
-where in the last step we use the condition (1.5). $\square$
-
-Approximate differential privacy
---------------------------------
-
-Unfortunately, $\epsilon$-dp does not apply to the most commonly used
-noise, the Gaussian noise. To fix this, we need to relax the definition
-a bit.
-
-**Definition**. A mechanism $M$ is said to be
-$(\epsilon, \delta)$*-differentially private* if for all $x, x' \in X$
-with $d(x, x') = 1$ and for all measureable $S \subset \mathbb R^d$
-
-$$\mathbb P(M(x) \in S) \le e^\epsilon P(M(x') \in S) + \delta. \qquad (2)$$
-
-Immediately we see that the $(\epsilon, \delta)$-dp is meaningful only
-if $\delta < 1$.
-
-### Indistinguishability
-
-To understand $(\epsilon, \delta)$-dp, it is helpful to study
-$(\epsilon, \delta)$-indistinguishability.
-
-**Definition**. Two probability measures $p$ and $q$ on
-the same space are called $(\epsilon, \delta)$*-ind(istinguishable)* if
-for all measureable sets $S$:
-
-$$\begin{aligned}
-p(S) \le e^\epsilon q(S) + \delta, \qquad (3) \\
-q(S) \le e^\epsilon p(S) + \delta. \qquad (4)
-\end{aligned}$$
-
-As before, we also call random variables $\xi$ and $\eta$ to be
-$(\epsilon, \delta)$-ind if their laws are $(\epsilon, \delta)$-ind.
-When $\delta = 0$, we call it $\epsilon$-ind.
-
-Immediately we have
-
-**Claim 0**. $M$ is $(\epsilon, \delta)$-dp (resp.
-$\epsilon$-dp) iff $M(x)$ and $M(x')$ are $(\epsilon, \delta)$-ind
-(resp. $\epsilon$-ind) for all $x$ and $x'$ with distance $1$.
-
-**Definition (Divergence
-Variable)**. Let $p$ and $q$ be two probability measures. Let $\xi$ be a
-random variable distributed according to $p$, we define a random
-variable $L(p || q)$ by
-
-$$L(p || q) := \log {p(\xi) \over q(\xi)},$$
-
-and call it the *divergence variable* of $(p, q)$.
-
-One interesting and readily verifiable fact is
-
-$$\mathbb E L(p || q) = D(p || q)$$
-
-where $D$ is the [KL-divergence](https://en.wikipedia.org/wiki/Kullback%E2%80%93Leibler_divergence).
-
-**Claim 1**. If
-
-$$\begin{aligned}
-\mathbb P(L(p || q) \le \epsilon) &\ge 1 - \delta, \qquad(5) \\
-\mathbb P(L(q || p) \le \epsilon) &\ge 1 - \delta
-\end{aligned}$$
-
-then $p$ and $q$ are $(\epsilon, \delta)$-ind.
-
-**Proof**. We verify (3), and (4) can be shown in the same
-way. Let $A := \{y \in Y: \log {p(y) \over q(y)} > \epsilon\}$, then by
-(5) we have
-
-$$p(A) < \delta.$$
-
-So
-
-$$p(S) = p(S \cap A) + p(S \setminus A) \le \delta + e^\epsilon q(S \setminus A) \le \delta + e^\epsilon q(S).$$
-
-$\square$
-
-This Claim translates differential privacy to the tail bound of
-divergence variables, and for the rest of this post all dp results are
-obtained by estimating this tail bound.
-
-In the following we discuss the converse of Claim 1. The discussions are
-rather technical, and readers can skip to the [next subsection](#back-to-approximate-differential-privacy) on first
-reading.
-
-The converse of Claim 1 is not true.
-
-**Claim 2**. There exists $\epsilon, \delta > 0$, and $p$
-and $q$ that are $(\epsilon, \delta)$-ind, such that
-
-$$\begin{aligned}
-\mathbb P(L(p || q) \le \epsilon) &< 1 - \delta, \\
-\mathbb P(L(q || p) \le \epsilon) &< 1 - \delta
-\end{aligned}$$
-
-**Proof**. Here\'s a example. Let $Y = \{0, 1\}$, and
-$p(0) = q(1) = 2 / 5$ and $p(1) = q(0) = 3 / 5$. Then it is not hard to
-verify that $p$ and $q$ are $(\log {4 \over 3}, {1 \over 3})$-ind: just
-check (3) for all four possible $S \subset Y$ and (4) holds by symmetry.
-On the other hand,
-
-$$\mathbb P(L(p || q) \le \log {4 \over 3}) = \mathbb P(L(q || p) \le \log {4 \over 3}) = {2 \over 5} < {2 \over 3}.$$
-
-$\square$
-
-A weaker version of the converse of Claim 1 is true
-(Kasiviswanathan-Smith 2015), though:
-
-**Claim 3**. Let $\alpha > 1$. If $p$ and $q$ are
-$(\epsilon, \delta)$-ind, then
-
-$$\mathbb P(L(p || q) > \alpha \epsilon) < {1 \over 1 - \exp((1 - \alpha) \epsilon)} \delta.$$
-
-**Proof**. Define
-
-$$S = \{y: p(y) > e^{\alpha \epsilon} q(y)\}.$$
-
-Then we have
-
-$$e^{\alpha \epsilon} q(S) < p(S) \le e^\epsilon q(S) + \delta,$$
-
-where the first inequality is due to the definition of $S$, and the
-second due to the $(\epsilon, \delta)$-ind. Therefore
-
-$$q(S) \le {\delta \over e^{\alpha \epsilon} - e^\epsilon}.$$
-
-Using the $(\epsilon, \delta)$-ind again we have
-
-$$p(S) \le e^\epsilon q(S) + \delta = {1 \over 1 - e^{(1 - \alpha) \epsilon}} \delta.$$
-
-$\square$
-
-This can be quite bad if $\epsilon$ is small.
-
-To prove the composition theorems in the next section, we need a
-condition better than that in Claim 1 so that we can go back and forth
-between indistinguishability and such condition. In other words, we need
-a *characterisation* of indistinguishability.
-
-Let us take a careful look at the condition in Claim 1 and call it
-**C1**:
-
-**C1**. $\mathbb P(L(p || q) \le \epsilon) \ge 1 - \delta$ and
-$\mathbb P(L(q || p) \le \epsilon) \ge 1 - \delta$
-
-It is equivalent to
-
-**C2**. there exist events $A, B \subset Y$ with probabilities
-$p(A)$ and $q(B)$ at least $1 - \delta$ such that
-$\log p(y) - \log q(y) \le \epsilon$ for all $y \in A$ and
-$\log q(y) - \log p(y) \le \epsilon$ for all $y \in B$.
-
-A similar-looking condition to **C2** is the following:
-
-**C3**. Let $\Omega$ be the [underlying probability
-space](https://en.wikipedia.org/wiki/Probability_space#Definition).
-There exist two events $E, F \subset \Omega$ with
-$\mathbb P(E), \mathbb P(F) \ge 1 - \delta$, such that
-$|\log p_{|E}(y) - \log q_{|F}(y)| \le \epsilon$ for all $y \in Y$.
-
-Here $p_{|E}$ (resp. $q_{|F}$) is $p$ (resp. $q$) conditioned on event
-$E$ (resp. $F$).
-
-**Remark**. Note that the events in **C2** and
-**C3** are in different spaces, and therefore we can not write
-$p_{|E}(S)$ as $p(S | E)$ or $q_{|F}(S)$ as $q(S | F)$. In fact, if we
-let $E$ and $F$ in **C3** be subsets of $Y$ with
-$p(E), q(F) \ge 1 - \delta$ and assume $p$ and $q$ have the same
-supports, then **C3** degenerates to a stronger condition than
-**C2**. Indeed, in this case $p_E(y) = p(y) 1_{y \in E}$ and
-$q_F(y) = q(y) 1_{y \in F}$, and so $p_E(y) \le e^\epsilon q_F(y)$
-forces $E \subset F$. We also obtain $F \subset E$ in the same way. This
-gives us $E = F$, and **C3** becomes **C2** with
-$A = B = E = F$.
-
-As it turns out, **C3** is the condition we need.
-
-**Claim 4**. Two probability measures $p$ and $q$ are
-$(\epsilon, \delta)$-ind if and only if **C3** holds.
-
-**Proof**(Murtagh-Vadhan 2018). The \"if\" direction is proved
-in the same way as Claim 1. Without loss of generality we may assume
-$\mathbb P(E) = \mathbb P(F) \ge 1 - \delta$. To see this, suppose $F$
-has higher probability than $E$, then we can substitute $F$ with a
-subset of $F$ that has the same probability as $E$ (with possible
-enlargement of the probability space).
-
-Let $\xi \sim p$ and $\eta \sim q$ be two independent random variables,
-then
-
-$$\begin{aligned}
-p(S) &= \mathbb P(\xi \in S | E) \mathbb P(E) + \mathbb P(\xi \in S; E^c) \\
-&\le e^\epsilon \mathbb P(\eta \in S | F) \mathbb P(E) + \delta \\
-&= e^\epsilon \mathbb P(\eta \in S | F) \mathbb P(F) + \delta\\
-&\le e^\epsilon q(S) + \delta.
-\end{aligned}$$
-
-The \"only-if\" direction is more involved.
-
-We construct events $E$ and $F$ by constructing functions
-$e, f: Y \to [0, \infty)$ satisfying the following conditions:
-
-1. $0 \le e(y) \le p(y)$ and $0 \le f(y) \le q(y)$ for all $y \in Y$.
-2. $|\log e(y) - \log f(y)| \le \epsilon$ for all $y \in Y$.
-3. $e(Y), f(Y) \ge 1 - \delta$.
-4. $e(Y) = f(Y)$.
-
-Here for a set $S \subset Y$, $e(S) := \int_S e(y) dy$, and the same
-goes for $f(S)$.
-
-Let $\xi \sim p$ and $\eta \sim q$. Then we define $E$ and $F$ by
-
-$$\mathbb P(E | \xi = y) = e(y) / p(y) \\
-\mathbb P(F | \eta = y) = f(y) / q(y).$$
-
-**Remark inside proof**. This can seem a bit
-confusing. Intuitively, we can think of it this way when $Y$ is finite:
-Recall a random variable on $Y$ is a function from the probability space
-$\Omega$ to $Y$. Let event $G_y \subset \Omega$ be defined as
-$G_y = \xi^{-1} (y)$. We cut $G_y$ into the disjoint union of $E_y$ and
-$G_y \setminus E_y$ such that $\mathbb P(E_y) = e(y)$. Then
-$E = \bigcup_{y \in Y} E_y$. So $e(y)$ can be seen as the \"density\" of
-$E$.
-
-Indeed, given $E$ and $F$ defined this way, we have
-
-$$p_E(y) = {e(y) \over e(Y)} \le {\exp(\epsilon) f(y) \over e(Y)} = {\exp(\epsilon) f(y) \over f(Y)} = \exp(\epsilon) q_F(y).$$
-
-and
-
-$$\mathbb P(E) = \int \mathbb P(E | \xi = y) p(y) dy = e(Y) \ge 1 - \delta,$$
-
-and the same goes for $\mathbb P(F)$.
-
-What remains is to construct $e(y)$ and $f(y)$ satisfying the four
-conditions.
-
-Like in the proof of Claim 1, let $S, T \subset Y$ be defined as
-
-$$\begin{aligned}
-S := \{y: p(y) > \exp(\epsilon) q(y)\},\\
-T := \{y: q(y) > \exp(\epsilon) p(y)\}.
-\end{aligned}$$
-
-Let
-
-$$\begin{aligned}
-e(y) &:= \exp(\epsilon) q(y) 1_{y \in S} + p(y) 1_{y \notin S}\\
-f(y) &:= \exp(\epsilon) p(y) 1_{y \in T} + q(y) 1_{y \notin T}. \qquad (6)
-\end{aligned}$$
-
-By checking them on the three disjoint subsets $S$, $T$, $(S \cup T)^c$,
-it is not hard to verify that the $e(y)$ and $f(y)$ constructed this way
-satisfy the first two conditions. They also satisfy the third condition:
-
-$$\begin{aligned}
-e(Y) &= 1 - (p(S) - \exp(\epsilon) q(S)) \ge 1 - \delta, \\
-f(Y) &= 1 - (q(T) - \exp(\epsilon) p(T)) \ge 1 - \delta.
-\end{aligned}$$
-
-If $e(Y) = f(Y)$ then we are done. Otherwise, without loss of
-generality, assume $e(Y) < f(Y)$, then all it remains to do is to reduce
-the value of $f(y)$ while preserving Condition 1, 2 and 3, until
-$f(Y) = e(Y)$.
-
-As it turns out, this can be achieved by reducing $f(y)$ on the set
-$\{y \in Y: q(y) > p(y)\}$. To see this, let us rename the $f(y)$
-defined in (6) $f_+(y)$, and construct $f_-(y)$ by
-
-$$f_-(y) := p(y) 1_{y \in T} + (q(y) \wedge p(y)) 1_{y \notin T}.$$
-
-It is not hard to show that not only $e(y)$ and $f_-(y)$ also satisfy
-conditions 1-3, but
-
-$$e(y) \ge f_-(y), \forall y \in Y,$$
-
-and thus $e(Y) \ge f_-(Y)$. Therefore there exists an $f$ that
-interpolates between $f_-$ and $f_+$ with $f(Y) = e(Y)$. $\square$
-
-To prove the adaptive composition theorem for approximate differential
-privacy, we need a similar claim (We use index shorthand
-$\xi_{< i} = \xi_{1 : i - 1}$ and similarly for other notations):
-
-**Claim 5**. Let $\xi_{1 : i}$ and $\eta_{1 : i}$ be random
-variables. Let
-
-$$\begin{aligned}
-p_i(S | y_{1 : i - 1}) := \mathbb P(\xi_i \in S | \xi_{1 : i - 1} = y_{1 : i - 1})\\
-q_i(S | y_{1 : i - 1}) := \mathbb P(\eta_i \in S | \eta_{1 : i - 1} = y_{1 : i - 1})
-\end{aligned}$$
-
-be the conditional laws of $\xi_i | \xi_{< i}$ and $\eta_i | \eta_{< i}$
-respectively. Then the following are equivalent:
-
-1. For any $y_{< i} \in Y^{i - 1}$, $p_i(\cdot | y_{< i})$ and
- $q_i(\cdot | y_{< i})$ are $(\epsilon, \delta)$-ind
-2. There exists events $E_i, F_i \subset \Omega$ with
- $\mathbb P(E_i | \xi_{<i} = y_{<i}) = \mathbb P(F_i | \eta_{<i} = y_{< i}) \ge 1 - \delta$
- for any $y_{< i}$, such that $p_{i | E_i}(\cdot | y_{< i})$ and
- $q_{i | E_i} (\cdot | y_{< i})$ are $\epsilon$-ind for any
- $y_{< i}$, where
- $$\begin{aligned}
- p_{i | E_i}(S | y_{1 : i - 1}) := \mathbb P(\xi_i \in S | E_i, \xi_{1 : i - 1} = y_{1 : i - 1})\\
- q_{i | F_i}(S | y_{1 : i - 1}) := \mathbb P(\eta_i \in S | F_i, \eta_{1 : i - 1} = y_{1 : i - 1})
- \end{aligned}$$
-
- are $p_i$ and $q_i$ conditioned on $E_i$ and $F_i$ respectively.
-
-**Proof**. Item 2 =\> Item 1: as in the Proof of Claim 4,
-
-$$\begin{aligned}
-p_i(S | y_{< i}) &= p_{i | E_i} (S | y_{< i}) \mathbb P(E_i | \xi_{< i} = y_{< i}) + p_{i | E_i^c}(S | y_{< i}) \mathbb P(E_i^c | \xi_{< i} = y_{< i}) \\
-&\le p_{i | E_i} (S | y_{< i}) \mathbb P(E_i | \xi_{< i} = y_{< i}) + \delta \\
-&= p_{i | E_i} (S | y_{< i}) \mathbb P(F_i | \xi_{< i} = y_{< i}) + \delta \\
-&\le e^\epsilon q_{i | F_i} (S | y_{< i}) \mathbb P(F_i | \xi_{< i} = y_{< i}) + \delta \\
-&= e^\epsilon q_i (S | y_{< i}) + \delta.
-\end{aligned}$$
-
-The direction from
-$q_i(S | y_{< i}) \le e^\epsilon p_i(S | y_{< i}) + \delta$ can be shown
-in the same way.
-
-Item 1 =\> Item 2: as in the Proof of Claim 4 we construct
-$e(y_{1 : i})$ and $f(y_{1 : i})$ as \"densities\" of events $E_i$ and
-$F_i$.
-
-Let
-
-$$\begin{aligned}
-e(y_{1 : i}) &:= e^\epsilon q_i(y_i | y_{< i}) 1_{y_i \in S_i(y_{< i})} + p_i(y_i | y_{< i}) 1_{y_i \notin S_i(y_{< i})}\\
-f(y_{1 : i}) &:= e^\epsilon p_i(y_i | y_{< i}) 1_{y_i \in T_i(y_{< i})} + q_i(y_i | y_{< i}) 1_{y_i \notin T_i(y_{< i})}\\
-\end{aligned}$$
-
-where
-
-$$\begin{aligned}
-S_i(y_{< i}) = \{y_i \in Y: p_i(y_i | y_{< i}) > e^\epsilon q_i(y_i | y_{< i})\}\\
-T_i(y_{< i}) = \{y_i \in Y: q_i(y_i | y_{< i}) > e^\epsilon p_i(y_i | y_{< i})\}.
-\end{aligned}$$
-
-Then $E_i$ and $F_i$ are defined as
-
-$$\begin{aligned}
-\mathbb P(E_i | \xi_{\le i} = y_{\le i}) &= {e(y_{\le i}) \over p_i(y_{\le i})},\\
-\mathbb P(F_i | \xi_{\le i} = y_{\le i}) &= {f(y_{\le i}) \over q_i(y_{\le i})}.
-\end{aligned}$$
-
-The rest of the proof is almost the same as the proof of Claim 4.
-$\square$
-
-### Back to approximate differential privacy
-
-By Claim 0 and 1 we have
-
-**Claim 6**. If for all $x, x' \in X$ with distance $1$
-
-$$\mathbb P(L(M(x) || M(x')) \le \epsilon) \ge 1 - \delta,$$
-
-then $M$ is $(\epsilon, \delta)$-dp.
-
-Note that in the literature the divergence variable $L(M(x) || M(x'))$
-is also called the *privacy loss*.
-
-By Claim 0 and Claim 4 we have
-
-**Claim 7**. $M$ is $(\epsilon, \delta)$-dp if and only if
-for every $x, x' \in X$ with distance $1$, there exist events
-$E, F \subset \Omega$ with $\mathbb P(E) = \mathbb P(F) \ge 1 - \delta$,
-$M(x) | E$ and $M(x') | F$ are $\epsilon$-ind.
-
-We can further simplify the privacy loss $L(M(x) || M(x'))$, by
-observing the translational and scaling invariance of $L(\cdot||\cdot)$:
-
-$$\begin{aligned}
-L(\xi || \eta) &\overset{d}{=} L(\alpha \xi + \beta || \alpha \eta + \beta), \qquad \alpha \neq 0. \qquad (6.1)
-\end{aligned}$$
-
-With this and the definition
-
-$$M(x) = f(x) + \zeta$$
-
-for some random variable $\zeta$, we have
-
-$$L(M(x) || M(x')) \overset{d}{=} L(\zeta || \zeta + f(x') - f(x)).$$
-
-Without loss of generality, we can consider $f$ with sensitivity $1$,
-for
-
-$$L(f(x) + S_f \zeta || f(x') + S_f \zeta) \overset{d}{=} L(S_f^{-1} f(x) + \zeta || S_f^{-1} f(x') + \zeta)$$
-
-so for any noise $\zeta$ that achieves $(\epsilon, \delta)$-dp for a
-function with sensitivity $1$, we have the same privacy guarantee by for
-an arbitrary function with sensitivity $S_f$ by adding a noise
-$S_f \zeta$.
-
-With Claim 6 we can show that the Gaussian mechanism is approximately
-differentially private. But first we need to define it.
-
-**Definition (Gaussian mechanism)**.
-Given a query $f: X \to Y$, the *Gaussian mechanism* $M$ adds a Gaussian
-noise to the query:
-
-$$M(x) = f(x) + N(0, \sigma^2 I).$$
-
-Some tail bounds for the Gaussian distribution will be useful.
-
-**Claim 8 (Gaussian tail bounds)**.
-Let $\xi \sim N(0, 1)$ be a standard normal distribution. Then for
-$t > 0$
-
-$$\mathbb P(\xi > t) < {1 \over \sqrt{2 \pi} t} e^{- {t^2 \over 2}}, \qquad (6.3)$$
-
-and
-
-$$\mathbb P(\xi > t) < e^{- {t^2 \over 2}}. \qquad (6.5)$$
-
-**Proof**. Both bounds are well known. The first can be proved
-using
-
-$$\int_t^\infty e^{- {y^2 \over 2}} dy < \int_t^\infty {y \over t} e^{- {y^2 \over 2}} dy.$$
-
-The second is shown using [Chernoff bound](https://en.wikipedia.org/wiki/Chernoff_bound). For any random variable $\xi$,
-
-$$\mathbb P(\xi > t) < {\mathbb E \exp(\lambda \xi) \over \exp(\lambda t)} = \exp(\kappa_\xi(\lambda) - \lambda t), \qquad (6.7)$$
-
-where $\kappa_\xi(\lambda) = \log \mathbb E \exp(\lambda \xi)$ is the
-cumulant of $\xi$. Since (6.7) holds for any $\lambda$, we can get the
-best bound by minimising $\kappa_\xi(\lambda) - \lambda t$ (a.k.a. the
-[Legendre transformation](https://en.wikipedia.org/wiki/Legendre_transformation)). When $\xi$ is standard normal, we get (6.5).
-$\square$
-
-**Remark**. We will use the Chernoff bound extensively in the
-second part of this post when considering Rényi differential privacy.
-
-**Claim 9**. The Gaussian mechanism on a query $f$ is
-$(\epsilon, \delta)$-dp, where
-
-$$\delta = \exp(- (\epsilon \sigma / S_f - (2 \sigma / S_f)^{-1})^2 / 2). \qquad (6.8)$$
-
-Conversely, to achieve give $(\epsilon, \delta)$-dp, we may set
-
-$$\sigma > \left(\epsilon^{-1} \sqrt{2 \log \delta^{-1}} + (2 \epsilon)^{- {1 \over 2}}\right) S_f \qquad (6.81)$$
-
-or
-
-$$\sigma > (\epsilon^{-1} (1 \vee \sqrt{(\log (2 \pi)^{-1} \delta^{-2})_+}) + (2 \epsilon)^{- {1 \over 2}}) S_f \qquad (6.82)$$
-
-or
-
-$$\sigma > \epsilon^{-1} \sqrt{\log e^\epsilon \delta^{-2}} S_f \qquad (6.83)$$
-
-or
-
-$$\sigma > \epsilon^{-1} (\sqrt{1 + \epsilon} \vee \sqrt{(\log e^\epsilon (2 \pi)^{-1} \delta^{-2})_+}) S_f. \qquad (6.84)$$
-
-**Proof**. As discussed before we only need to consider the
-case where $S_f = 1$. Fix arbitrary $x, x' \in X$ with $d(x, x') = 1$.
-Let $\zeta = (\zeta_1, ..., \zeta_d) \sim N(0, I_d)$.
-
-By Claim 6 it suffices to bound
-
-$$\mathbb P(L(M(x) || M(x')) > \epsilon)$$
-
-We have by the linear invariance of $L$,
-
-$$L(M(x) || M(x')) = L(f(x) + \sigma \zeta || f(x') + \sigma \zeta) \overset{d}{=} L(\zeta|| \zeta + \Delta / \sigma),$$
-
-where $\Delta := f(x') - f(x)$.
-
-Plugging in the Gaussian density, we have
-
-$$L(M(x) || M(x')) \overset{d}{=} \sum_i {\Delta_i \over \sigma} \zeta_i + \sum_i {\Delta_i^2 \over 2 \sigma^2} \overset{d}{=} {\|\Delta\|_2 \over \sigma} \xi + {\|\Delta\|_2^2 \over 2 \sigma^2}.$$
-
-where $\xi \sim N(0, 1)$.
-
-Hence
-
-$$\mathbb P(L(M(x) || M(x')) > \epsilon) = \mathbb P(\zeta > {\sigma \over \|\Delta\|_2} \epsilon - {\|\Delta\|_2 \over 2 \sigma}).$$
-
-Since $\|\Delta\|_2 \le S_f = 1$, we have
-
-$$\mathbb P(L(M(x) || M(x')) > \epsilon) \le \mathbb P(\xi > \sigma \epsilon - (2 \sigma)^{-1}).$$
-
-Thus the problem is reduced to the tail bound of a standard normal
-distribution, so we can use Claim 8. Note that we implicitly require
-$\sigma > (2 \epsilon)^{- 1 / 2}$ here so that
-$\sigma \epsilon - (2 \sigma)^{-1} > 0$ and we can use the tail bounds.
-
-Using (6.3) we have
-
-$$\mathbb P(L(M(x) || M(x')) > \epsilon) < \exp(- (\epsilon \sigma - (2 \sigma)^{-1})^2 / 2).$$
-
-This gives us (6.8).
-
-To bound the right hand by $\delta$, we require
-
-$$\epsilon \sigma - {1 \over 2 \sigma} > \sqrt{2 \log \delta^{-1}}. \qquad (6.91)$$
-
-Solving this inequality we have
-
-$$\sigma > {\sqrt{2 \log \delta^{-1}} + \sqrt{2 \log \delta^{-1} + 2 \epsilon} \over 2 \epsilon}.$$
-
-Using
-$\sqrt{2 \log \delta^{-1} + 2 \epsilon} \le \sqrt{2 \log \delta^{-1}} + \sqrt{2 \epsilon}$,
-we can achieve the above inequality by having
-
-$$\sigma > \epsilon^{-1} \sqrt{2 \log \delta^{-1}} + (2 \epsilon)^{-{1 \over 2}}.$$
-
-This gives us (6.81).
-
-Alternatively, we can use the concavity of $\sqrt{\cdot}$:
-
-$$(2 \epsilon)^{-1} (\sqrt{2 \log \delta^{-1}} + \sqrt{2 \log \delta^{-1} + 2 \epsilon}) \le \epsilon^{-1} \sqrt{\log e^\epsilon \delta^{-2}},$$
-
-which gives us (6.83)
-
-Back to (6.9), if we use (6.5) instead, we need
-
-$$\log t + {t^2 \over 2} > \log {(2 \pi)^{- 1 / 2} \delta^{-1}}$$
-
-where $t = \epsilon \sigma - (2 \sigma)^{-1}$. This can be satisfied if
-
-$$\begin{aligned}
-t &> 1 \qquad (6.93)\\
-t &> \sqrt{\log (2 \pi)^{-1} \delta^{-2}}. \qquad (6.95)
-\end{aligned}$$
-
-We can solve both inequalities as before and obtain
-
-$$\sigma > \epsilon^{-1} (1 \vee \sqrt{(\log (2 \pi)^{-1} \delta^{-2})_+}) + (2 \epsilon)^{- {1 \over 2}},$$
-
-or
-
-$$\sigma > \epsilon^{-1}(\sqrt{1 + \epsilon} \vee \sqrt{(\log e^\epsilon (2 \pi)^{-1} \delta^{-2})_+}).$$
-
-This gives us (6.82)(6.84). $\square$
-
-When $\epsilon \le \alpha$ is bounded, by (6.83) (6.84) we can require
-either
-
-$$\sigma > \epsilon^{-1} (\sqrt{\log e^\alpha \delta^{-2}}) S_f$$
-
-or
-
-$$\sigma > \epsilon^{-1} (\sqrt{1 + \alpha} \vee \sqrt{(\log (2 \pi)^{-1} e^\alpha \delta^{-2})_+}) S_f.$$
-
-The second bound is similar to and slightly better than the one in
-Theorem A.1 of Dwork-Roth 2013, where $\alpha = 1$:
-
-$$\sigma > \epsilon^{-1} \left({3 \over 2} \vee \sqrt{(2 \log {5 \over 4} \delta^{-1})_+}\right) S_f.$$
-
-Note that the lower bound of ${3 \over 2}$ is implicitly required in the
-proof of Theorem A.1.
-
-Composition theorems
---------------------
-
-So far we have seen how a mechanism made of a single query plus a noise
-can be proved to be differentially private. But we need to understand
-the privacy when composing several mechanisms, combinatorially or
-sequentially. Let us first define the combinatorial case:
-
-**Definition (Independent
-composition)**. Let $M_1, ..., M_k$ be $k$ mechanisms with independent
-noises. The mechanism $M = (M_1, ..., M_k)$ is called the *independent
-composition* of $M_{1 : k}$.
-
-To define the adaptive composition, let us motivate it with an example
-of gradient descent. Consider the loss function $\ell(x; \theta)$ of a neural network,
-where $\theta$ is the parameter and $x$ the input, gradient descent updates
-its parameter $\theta$ at each time $t$:
-
-$$\theta_{t} = \theta_{t - 1} - \alpha m^{-1} \sum_{i = 1 : m} \nabla_\theta \ell(x_i; \theta) |_{\theta = \theta_{t - 1}}.$$
-
-We may add privacy by adding noise $\zeta_t$ at each step:
-
-$$\theta_{t} = \theta_{t - 1} - \alpha m^{-1} \sum_{i = 1 : m} \nabla_\theta \ell(x_i; \theta) |_{\theta = \theta_{t - 1}} + \zeta_t. \qquad (6.97)$$
-
-Viewed as a sequence of mechanism, we have that at each time $t$, the
-mechanism $M_t$ takes input $x$, and outputs $\theta_t$. But $M_t$ also
-depends on the output of the previous mechanism $M_{t - 1}$. To this
-end, we define the adaptive composition.
-
-**Definition (Adaptive
-composition)**. Let $({M_i(y_{1 : i - 1})})_{i = 1 : k}$ be $k$
-mechanisms with independent noises, where $M_1$ has no parameter, $M_2$
-has one parameter in $Y$, $M_3$ has two parameters in $Y$ and so on. For
-$x \in X$, define $\xi_i$ recursively by
-
-$$\begin{aligned}
-\xi_1 &:= M_1(x)\\
-\xi_i &:= M_i(\xi_1, \xi_2, ..., \xi_{i - 1}) (x).
-\end{aligned}$$
-
-The *adaptive composition* of $M_{1 : k}$ is defined by
-$M(x) := (\xi_1, \xi_2, ..., \xi_k)$.
-
-The definition of adaptive composition may look a bit complicated, but
-the point is to describe $k$ mechanisms such that for each $i$, the
-output of the first, second, \..., $i - 1$th mechanisms determine the
-$i$th mechanism, like in the case of gradient descent.
-
-It is not hard to write down the differentially private gradient descent
-as a sequential composition:
-
-$$M_t(\theta_{1 : t - 1})(x) = \theta_{t - 1} - \alpha m^{-1} \sum_{i = 1 : m} \nabla_\theta \ell(x_i; \theta) |_{\theta = \theta_{t - 1}} + \zeta_t.$$
-
-In Dwork-Rothblum-Vadhan 2010 (see also Dwork-Roth 2013) the adaptive
-composition is defined in a more general way, but the definition is
-based on the same principle, and proofs in this post on adaptive
-compositions carry over.
-
-It is not hard to see that the adaptive composition degenerates to
-independent composition when each $M_i(y_{1 : i})$ evaluates to the same
-mechanism regardless of $y_{1 : i}$, in which case the $\xi_i$s are
-independent.
-
-In the following when discussing adaptive compositions we sometimes omit
-the parameters for convenience without risk of ambiguity, and write
-$M_i(y_{1 : i})$ as $M_i$, but keep in mind of the dependence on the
-parameters.
-
-It is time to state and prove the composition theorems. In this section
-we consider $2 \times 2 \times 2 = 8$ cases, i.e. situations of three
-dimensions, where there are two choices in each dimension:
-
-1. Composition of $\epsilon$-dp or more generally
- $(\epsilon, \delta)$-dp mechanisms
-2. Composition of independent or more generally adaptive mechanisms
-3. Basic or advanced compositions
-
-Note that in the first two dimensions the second choice is more general
-than the first.
-
-The proofs of these composition theorems will be laid out as follows:
-
-1. Claim 10 - Basic composition theorem for $(\epsilon, \delta)$-dp
- with adaptive mechanisms: by a direct proof with an induction
- argument
-2. Claim 14 - Advanced composition theorem for $\epsilon$-dp with
- independent mechanisms: by factorising privacy loss and using
- [Hoeffding\'s Inequality](https://en.wikipedia.org/wiki/Hoeffding%27s_inequality)
-3. Claim 16 - Advanced composition theorem for $\epsilon$-dp with
- adaptive mechanisms: by factorising privacy loss and using
- [Azuma\'s Inequality](https://en.wikipedia.org/wiki/Azuma%27s_inequality)
-4. Claims 17 and 18 - Advanced composition theorem for
- $(\epsilon, \delta)$-dp with independent / adaptive mechanisms: by
- using characterisations of $(\epsilon, \delta)$-dp in Claims 4 and 5
- as an approximation of $\epsilon$-dp and then using Proofs in Item 2
- or 3.
-
-**Claim 10 (Basic composition
-theorem).** Let $M_{1 : k}$ be $k$ mechanisms with independent noises
-such that for each $i$ and $y_{1 : i - 1}$, $M_i(y_{1 : i - 1})$ is
-$(\epsilon_i, \delta_i)$-dp. Then the adpative composition of
-$M_{1 : k}$ is $(\sum_i \epsilon_i, \sum_i \delta_i)$-dp.
-
-**Proof
-(Dwork-Lei 2009, see also Dwork-Roth 2013 Appendix B.1)**. Let $x$ and
-$x'$ be neighbouring points in $X$. Let $M$ be the adaptive composition
-of $M_{1 : k}$. Define
-
-$$\xi_{1 : k} := M(x), \qquad \eta_{1 : k} := M(x').$$
-
-Let $p^i$ and $q^i$ be the laws of $(\xi_{1 : i})$ and $(\eta_{1 : i})$
-respectively.
-
-Let $S_1, ..., S_k \subset Y$ and $T_i := \prod_{j = 1 : i} S_j$. We use
-two tricks.
-
-1. Since $\xi_i | \xi_{< i} = y_{< i}$ and
- $\eta_i | \eta_{< i} = y_{< i}$ are $(\epsilon_i, \delta_i)$-ind,
- and a probability is no greater than $1$,
- $$\begin{aligned}
- \mathbb P(\xi_i \in S_i | \xi_{< i} = y_{< i}) &\le (e^{\epsilon_i} \mathbb P(\eta_i \in S_i | \eta_{< i} = y_{< i}) + \delta_i) \wedge 1 \\
- &\le (e^{\epsilon_i} \mathbb P(\eta_i \in S_i | \eta_{< i} = y_{< i}) + \delta_i) \wedge (1 + \delta_i) \\
- &= (e^{\epsilon_i} \mathbb P(\eta_i \in S_i | \eta_{< i} = y_{< i}) \wedge 1) + \delta_i
- \end{aligned}$$
-
-2. Given $p$ and $q$ that are $(\epsilon, \delta)$-ind, define
- $$\mu(x) = (p(x) - e^\epsilon q(x))_+.$$
-
- We have
- $$\mu(S) \le \delta, \forall S$$
-
- In the following we define
- $\mu^{i - 1} = (p^{i - 1} - e^\epsilon q^{i - 1})_+$ for the same
- purpose.
-
-We use an inductive argument to prove the theorem:
-
-$$\begin{aligned}
-\mathbb P(\xi_{\le i} \in T_i) &= \int_{T_{i - 1}} \mathbb P(\xi_i \in S_i | \xi_{< i} = y_{< i}) p^{i - 1} (y_{< i}) dy_{< i} \\
-&\le \int_{T_{i - 1}} (e^{\epsilon_i} \mathbb P(\eta_i \in S_i | \eta_{< i} = y_{< i}) \wedge 1) p^{i - 1}(y_{< i}) dy_{< i} + \delta_i\\
-&\le \int_{T_{i - 1}} (e^{\epsilon_i} \mathbb P(\eta_i \in S_i | \eta_{< i} = y_{< i}) \wedge 1) (e^{\epsilon_1 + ... + \epsilon_{i - 1}} q^{i - 1}(y_{< i}) + \mu^{i - 1} (y_{< i})) dy_{< i} + \delta_i\\
-&\le \int_{T_{i - 1}} e^{\epsilon_i} \mathbb P(\eta_i \in S_i | \eta_{< i} = y_{< i}) e^{\epsilon_1 + ... + \epsilon_{i - 1}} q^{i - 1}(y_{< i}) dy_{< i} + \mu_{i - 1}(T_{i - 1}) + \delta_i\\
-&\le e^{\epsilon_1 + ... + \epsilon_i} \mathbb P(\eta_{\le i} \in T_i) + \delta_1 + ... + \delta_{i - 1} + \delta_i.\\
-\end{aligned}$$
-
-In the second line we use Trick 1; in the third line we use the
-induction assumption; in the fourth line we multiply the first term in
-the first braket with first term in the second braket, and the second
-term (i.e. $1$) in the first braket with the second term in the second
-braket (i.e. the $\mu$ term); in the last line we use Trick 2.
-
-The base case $i = 1$ is true since $M_1$ is
-$(\epsilon_1, \delta_1)$-dp. $\square$
-
-To prove the advanced composition theorem, we start with some lemmas.
-
-**Claim 11**. If $p$ and $q$ are $\epsilon$-ind, then
-
-$$D(p || q) + D(q || p) \le \epsilon(e^\epsilon - 1).$$
-
-**Proof**. Since $p$ and $q$ are $\epsilon$-ind, we have
-$|\log p(x) - \log q(x)| \le \epsilon$ for all $x$. Let
-$S := \{x: p(x) > q(x)\}$. Then we have on
-
-$$\begin{aligned}
-D(p || q) + D(q || p) &= \int (p(x) - q(x)) (\log p(x) - \log q(x)) dx\\
-&= \int_S (p(x) - q(x)) (\log p(x) - \log q(x)) dx + \int_{S^c} (q(x) - p(x)) (\log q(x) - \log p(x)) dx\\
-&\le \epsilon(\int_S p(x) - q(x) dx + \int_{S^c} q(x) - p(x) dx)
-\end{aligned}$$
-
-Since on $S$ we have $q(x) \le p(x) \le e^\epsilon q(x)$, and on $S^c$
-we have $p(x) \le q(x) \le e^\epsilon p(x)$, we obtain
-
-$$D(p || q) + D(q || p) \le \epsilon \int_S (1 - e^{-\epsilon}) p(x) dx + \epsilon \int_{S^c} (e^{\epsilon} - 1) p(x) dx \le \epsilon (e^{\epsilon} - 1),$$
-
-where in the last step we use $e^\epsilon - 1 \ge 1 - e^{- \epsilon}$
-and $p(S) + p(S^c) = 1$. $\square$
-
-**Claim 12**. If $p$ and $q$ are $\epsilon$-ind, then
-
-$$D(p || q) \le a(\epsilon) \ge D(q || p),$$
-
-where
-
-$$a(\epsilon) = \epsilon (e^\epsilon - 1) 1_{\epsilon \le \log 2} + \epsilon 1_{\epsilon > \log 2} \le (\log 2)^{-1} \epsilon^2 1_{\epsilon \le \log 2} + \epsilon 1_{\epsilon > \log 2}. \qquad (6.98)$$
-
-**Proof**. Since $p$ and $q$ are $\epsilon$-ind, we have
-
-$$D(p || q) = \mathbb E_{\xi \sim p} \log {p(\xi) \over q(\xi)} \le \max_y {\log p(y) \over \log q(y)} \le \epsilon.$$
-
-Comparing the quantity in Claim 11 ($\epsilon(e^\epsilon - 1)$) with the
-quantity above ($\epsilon$), we arrive at the conclusion. $\square$
-
-**Claim 13 ([Hoeffding\'s Inequality](https://en.wikipedia.org/wiki/Hoeffding%27s_inequality))**.
-Let $L_i$ be independent random variables with
-$|L_i| \le b$, and let $L = L_1 + ... + L_k$, then for $t > 0$,
-
-$$\mathbb P(L - \mathbb E L \ge t) \le \exp(- {t^2 \over 2 k b^2}).$$
-
-**Claim 14
-(Advanced Independent Composition Theorem)** ($\delta = 0$). Fix
-$0 < \beta < 1$. Let $M_1, ..., M_k$ be $\epsilon$-dp, then the
-independent composition $M$ of $M_{1 : k}$ is
-$(k a(\epsilon) + \sqrt{2 k \log \beta^{-1}} \epsilon, \beta)$-dp.
-
-**Remark**. By (6.98) we know that
-$k a(\epsilon) + \sqrt{2 k \log \beta^{-1}} \epsilon = \sqrt{2 k \log \beta^{-1}} \epsilon + k O(\epsilon^2)$
-when $\epsilon$ is sufficiently small, in which case the leading term is of order
-$O(\sqrt k \epsilon)$ and we save a $\sqrt k$ in the $\epsilon$-part
-compared to the Basic Composition Theorem (Claim 10).
-
-**Remark**. In practice one can try different choices of $\beta$ and settle
-with the one that gives the best privacy guarantee.
-See the discussions at the end of [Part 2 of this post](/posts/2019-03-14-great-but-manageable-expectations.html).
-
-**Proof**. Let $p_i$, $q_i$, $p$ and $q$ be the laws of
-$M_i(x)$, $M_i(x')$, $M(x)$ and $M(x')$ respectively.
-
-$$\mathbb E L_i = D(p_i || q_i) \le a(\epsilon),$$
-
-where $L_i := L(p_i || q_i)$. Due to $\epsilon$-ind also have
-
-$$|L_i| \le \epsilon.$$
-
-Therefore, by Hoeffding\'s Inequality,
-
-$$\mathbb P(L - k a(\epsilon) \ge t) \le \mathbb P(L - \mathbb E L \ge t) \le \exp(- t^2 / 2 k \epsilon^2),$$
-
-where $L := \sum_i L_i = L(p || q)$.
-
-Plugging in $t = \sqrt{2 k \epsilon^2 \log \beta^{-1}}$, we have
-
-$$\mathbb P(L(p || q) \le k a(\epsilon) + \sqrt{2 k \epsilon^2 \log \beta^{-1}}) \ge 1 - \beta.$$
-
-Similarly we also have
-
-$$\mathbb P(L(q || p) \le k a(\epsilon) + \sqrt{2 k \epsilon^2 \log \beta^{-1}}) \ge 1 - \beta.$$
-
-By Claim 1 we arrive at the conclusion. $\square$
-
-**Claim 15 ([Azuma\'s Inequality](https://en.wikipedia.org/wiki/Azuma%27s_inequality))**.
-Let $X_{0 : k}$ be a [supermartingale](https://en.wikipedia.org/wiki/Martingale_(probability_theory)).
-If $|X_i - X_{i - 1}| \le b$, then
-
-$$\mathbb P(X_k - X_0 \ge t) \le \exp(- {t^2 \over 2 k b^2}).$$
-
-Azuma\'s Inequality implies a slightly weaker version of Hoeffding\'s
-Inequality. To see this, let $L_{1 : k}$ be independent variables with
-$|L_i| \le b$. Let $X_i = \sum_{j = 1 : i} L_j - \mathbb E L_j$. Then
-$X_{0 : k}$ is a martingale, and
-
-$$| X_i - X_{i - 1} | = | L_i - \mathbb E L_i | \le 2 b,$$
-
-since $\|L_i\|_1 \le \|L_i\|_\infty$. Hence by Azuma\'s Inequality,
-
-$$\mathbb P(L - \mathbb E L \ge t) \le \exp(- {t^2 \over 8 k b^2}).$$
-
-Of course here we have made no assumption on $\mathbb E L_i$. If instead
-we have some bound for the expectation, say $|\mathbb E L_i| \le a$,
-then by the same derivation we have
-
-$$\mathbb P(L - \mathbb E L \ge t) \le \exp(- {t^2 \over 2 k (a + b)^2}).$$
-
-It is not hard to see what Azuma is to Hoeffding is like adaptive
-composition to independent composition. Indeed, we can use Azuma\'s
-Inequality to prove the Advanced Adaptive Composition Theorem for
-$\delta = 0$.
-
-**Claim 16
-(Advanced Adaptive Composition Theorem)** ($\delta = 0$). Let
-$\beta > 0$. Let $M_{1 : k}$ be $k$ mechanisms with independent noises
-such that for each $i$ and $y_{1 : i}$, $M_i(y_{1 : i})$ is
-$(\epsilon, 0)$-dp. Then the adpative composition of $M_{1 : k}$ is
-$(k a(\epsilon) + \sqrt{2 k \log \beta^{-1}} (\epsilon + a(\epsilon)), \beta)$-dp.
-
-**Proof**. As before, let $\xi_{1 : k} \overset{d}{=} M(x)$
-and $\eta_{1 : k} \overset{d}{=} M(x')$, where $M$ is the adaptive
-composition of $M_{1 : k}$. Let $p_i$ (resp. $q_i$) be the law of
-$\xi_i | \xi_{< i}$ (resp. $\eta_i | \eta_{< i}$). Let $p^i$ (resp.
-$q^i$) be the law of $\xi_{\le i}$ (resp. $\eta_{\le i}$). We want to
-construct supermartingale $X$. To this end, let
-
-$$X_i = \log {p^i(\xi_{\le i}) \over q^i(\xi_{\le i})} - i a(\epsilon) $$
-
-We show that $(X_i)$ is a supermartingale:
-
-$$\begin{aligned}
-\mathbb E(X_i - X_{i - 1} | X_{i - 1}) &= \mathbb E \left(\log {p_i (\xi_i | \xi_{< i}) \over q_i (\xi_i | \xi_{< i})} - a(\epsilon) | \log {p^{i - 1} (\xi_{< i}) \over q^{i - 1} (\xi_{< i})}\right) \\
-&= \mathbb E \left( \mathbb E \left(\log {p_i (\xi_i | \xi_{< i}) \over q_i (\xi_i | \xi_{< i})} | \xi_{< i}\right) | \log {p^{i - 1} (\xi_{< i}) \over q^{i - 1} (\xi_{< i})}\right) - a(\epsilon) \\
-&= \mathbb E \left( D(p_i (\cdot | \xi_{< i}) || q_i (\cdot | \xi_{< i})) | \log {p^{i - 1} (\xi_{< i}) \over q^{i - 1} (\xi_{< i})}\right) - a(\epsilon) \\
-&\le 0,
-\end{aligned}$$
-
-since by Claim 12
-$D(p_i(\cdot | y_{< i}) || q_i(\cdot | y_{< i})) \le a(\epsilon)$ for
-all $y_{< i}$.
-
-Since
-
-$$| X_i - X_{i - 1} | = | \log {p_i(\xi_i | \xi_{< i}) \over q_i(\xi_i | \xi_{< i})} - a(\epsilon) | \le \epsilon + a(\epsilon),$$
-
-by Azuma\'s Inequality,
-
-$$\mathbb P(\log {p^k(\xi_{1 : k}) \over q^k(\xi_{1 : k})} \ge k a(\epsilon) + t) \le \exp(- {t^2 \over 2 k (\epsilon + a(\epsilon))^2}). \qquad(6.99)$$
-
-Let $t = \sqrt{2 k \log \beta^{-1}} (\epsilon + a(\epsilon))$ we are
-done. $\square$
-
-**Claim 17
-(Advanced Independent Composition Theorem)**. Fix $0 < \beta < 1$. Let
-$M_1, ..., M_k$ be $(\epsilon, \delta)$-dp, then the independent
-composition $M$ of $M_{1 : k}$ is
-$(k a(\epsilon) + \sqrt{2 k \log \beta^{-1}} \epsilon, k \delta + \beta)$-dp.
-
-**Proof**. By Claim 4, there exist events $E_{1 : k}$ and
-$F_{1 : k}$ such that
-
-1. The laws $p_{i | E_i}$ and $q_{i | F_i}$ are $\epsilon$-ind.
-2. $\mathbb P(E_i), \mathbb P(F_i) \ge 1 - \delta$.
-
-Let $E := \bigcap E_i$ and $F := \bigcap F_i$, then they both have
-probability at least $1 - k \delta$, and $p_{i | E}$ and $q_{i | F}$ are
-$\epsilon$-ind.
-
-By Claim 14, $p_{|E}$ and $q_{|F}$ are
-$(\epsilon' := k a(\epsilon) + \sqrt{2 k \epsilon^2 \log \beta^{-1}}, \beta)$-ind.
-Let us shrink the bigger event between $E$ and $F$ so that they have
-equal probabilities. Then
-
-$$\begin{aligned}
-p (S) &\le p_{|E}(S) \mathbb P(E) + \mathbb P(E^c) \\
-&\le (e^{\epsilon'} q_{|F}(S) + \beta) \mathbb P(F) + k \delta\\
-&\le e^{\epsilon'} q(S) + \beta + k \delta.
-\end{aligned}$$
-
-$\square$
-
-**Claim 18
-(Advanced Adaptive Composition Theorem)**. Fix $0 < \beta < 1$. Let
-$M_{1 : k}$ be $k$ mechanisms with independent noises such that for each
-$i$ and $y_{1 : i}$, $M_i(y_{1 : i})$ is $(\epsilon, \delta)$-dp. Then
-the adpative composition of $M_{1 : k}$ is
-$(k a(\epsilon) + \sqrt{2 k \log \beta^{-1}} (\epsilon + a(\epsilon)), \beta + k \delta)$-dp.
-
-**Remark**.
-This theorem appeared in Dwork-Rothblum-Vadhan 2010, but I could not find a proof there.
-A proof can be found in Dwork-Roth 2013 (See Theorem 3.20 there).
-Here I prove it in a similar way, except that instead of the use of an intermediate random variable there,
-I use the conditional probability results from Claim 5, the approach mentioned in Vadhan 2017.
-
-**Proof**. By Claim 5, there exist events $E_{1 : k}$ and
-$F_{1 : k}$ such that
-
-1. The laws $p_{i | E_i}(\cdot | y_{< i})$ and
- $q_{i | F_i}(\cdot | y_{< i})$ are $\epsilon$-ind for all $y_{< i}$.
-2. $\mathbb P(E_i | y_{< i}), \mathbb P(F_i | y_{< i}) \ge 1 - \delta$
- for all $y_{< i}$.
-
-Let $E := \bigcap E_i$ and $F := \bigcap F_i$, then they both have
-probability at least $1 - k \delta$, and $p_{i | E}(\cdot | y_{< i}$ and
-$q_{i | F}(\cdot | y_{< i})$ are $\epsilon$-ind.
-
-By Advanced Adaptive Composition Theorem ($\delta = 0$), $p_{|E}$ and
-$q_{|F}$ are
-$(\epsilon' := k a(\epsilon) + \sqrt{2 k \log \beta^{-1}} (\epsilon + a(\epsilon)), \beta)$-ind.
-
-The rest is the same as in the proof of Claim 17. $\square$
-
-Subsampling
------------
-
-Stochastic gradient descent is like gradient descent, but with random
-subsampling.
-
-Recall we have been considering databases in the space $Z^m$. Let
-$n < m$ be a positive integer,
-$\mathcal I := \{I \subset [m]: |I| = n\}$ be the set of subsets of
-$[m]$ of size $n$, and $\gamma$ a random subset sampled uniformly from
-$\mathcal I$. Let $r = {n \over m}$ which we call the subsampling rate.
-Then we may add a subsampling module to the noisy gradient descent
-algorithm (6.97) considered before
-
-$$\theta_{t} = \theta_{t - 1} - \alpha n^{-1} \sum_{i \in \gamma} \nabla_\theta h_\theta(x_i) |_{\theta = \theta_{t - 1}} + \zeta_t. \qquad (7)$$
-
-It turns out subsampling has an amplification effect on privacy.
-
-**Claim 19 (Ullman 2017)**. Fix
-$r \in [0, 1]$. Let $n \le m$ be two nonnegative integers with
-$n = r m$. Let $N$ be an $(\epsilon, \delta)$-dp mechanism on $Z^n$.
-Define mechanism $M$ on $Z^m$ by
-
-$$M(x) = N(x_\gamma)$$
-
-Then $M$ is $(\log (1 + r(e^\epsilon - 1)), r \delta)$-dp.
-
-**Remark**. Some seem to cite
-Kasiviswanathan-Lee-Nissim-Raskhodnikova-Smith 2005 for this result, but
-it is not clear to me how it appears there.
-
-**Proof**. Let $x, x' \in Z^n$ such that they differ by one
-row $x_i \neq x_i'$. Naturally we would like to consider the cases where
-the index $i$ is picked and the ones where it is not separately. Let
-$\mathcal I_\in$ and $\mathcal I_\notin$ be these two cases:
-
-$$\begin{aligned}
-\mathcal I_\in = \{J \subset \mathcal I: i \in J\}\\
-\mathcal I_\notin = \{J \subset \mathcal I: i \notin J\}\\
-\end{aligned}$$
-
-We will use these notations later. Let $A$ be the event
-$\{\gamma \ni i\}$.
-
-Let $p$ and $q$ be the laws of $M(x)$ and $M(x')$ respectively. We
-collect some useful facts about them. First due to $N$ being
-$(\epsilon, \delta)$-dp,
-
-$$p_{|A}(S) \le e^\epsilon q_{|A}(S) + \delta.$$
-
-Also,
-
-$$p_{|A}(S) \le e^\epsilon p_{|A^c}(S) + \delta.$$
-
-To see this, note that being conditional laws, $p_A$ and $p_{A^c}$ are
-averages of laws over $\mathcal I_\in$ and $\mathcal I_\notin$
-respectively:
-
-$$\begin{aligned}
-p_{|A}(S) = |\mathcal I_\in|^{-1} \sum_{I \in \mathcal I_\in} \mathbb P(N(x_I) \in S)\\
-p_{|A^c}(S) = |\mathcal I_\notin|^{-1} \sum_{J \in \mathcal I_\notin} \mathbb P(N(x_J) \in S).
-\end{aligned}$$
-
-Now we want to pair the $I$\'s in $\mathcal I_\in$ and $J$\'s in
-$\mathcal I_\notin$ so that they differ by one index only, which means
-$d(x_I, x_J) = 1$. Formally, this means we want to consider the set:
-
-$$\mathcal D := \{(I, J) \in \mathcal I_\in \times \mathcal I_\notin: |I \cap J| = n - 1\}.$$
-
-We may observe by trying out some simple cases that every
-$I \in \mathcal I_\in$ is paired with $n$ elements in
-$\mathcal I_\notin$, and every $J \in \mathcal I_\notin$ is paired with
-$m - n$ elements in $\mathcal I_\in$. Therefore
-
-$$p_{|A}(S) = |\mathcal D|^{-1} \sum_{(I, J) \in \mathcal D} \mathbb P(N(x_I \in S)) \le |\mathcal D|^{-1} \sum_{(I, J) \in \mathcal D} (e^\epsilon \mathbb P(N(x_J \in S)) + \delta) = e^\epsilon p_{|A^c} (S) + \delta.$$
-
-Since each of the $m$ indices is picked independently with probability
-$r$, we have
-
-$$\mathbb P(A) = r.$$
-
-Let $t \in [0, 1]$ to be determined. We may write
-
-$$\begin{aligned}
-p(S) &= r p_{|A} (S) + (1 - r) p_{|A^c} (S)\\
-&\le r(t e^\epsilon q_{|A}(S) + (1 - t) e^\epsilon q_{|A^c}(S) + \delta) + (1 - r) q_{|A^c} (S)\\
-&= rte^\epsilon q_{|A}(S) + (r(1 - t) e^\epsilon + (1 - r)) q_{|A^c} (S) + r \delta\\
-&= te^\epsilon r q_{|A}(S) + \left({r \over 1 - r}(1 - t) e^\epsilon + 1\right) (1 - r) q_{|A^c} (S) + r \delta \\
-&\le \left(t e^\epsilon \wedge \left({r \over 1 - r} (1 - t) e^\epsilon + 1\right)\right) q(S) + r \delta. \qquad (7.5)
-\end{aligned}$$
-
-We can see from the last line that the best bound we can get is when
-
-$$t e^\epsilon = {r \over 1 - r} (1 - t) e^\epsilon + 1.$$
-
-Solving this equation we obtain
-
-$$t = r + e^{- \epsilon} - r e^{- \epsilon}$$
-
-and plugging this in (7.5) we have
-
-$$p(S) \le (1 + r(e^\epsilon - 1)) q(S) + r \delta.$$
-
-$\square$
-
-Since $\log (1 + x) < x$ for $x > 0$, we can rewrite the conclusion of
-the Claim to $(r(e^\epsilon - 1), r \delta)$-dp. Further more, if
-$\epsilon < \alpha$ for some $\alpha$, we can rewrite it as
-$(r \alpha^{-1} (e^\alpha - 1) \epsilon, r \delta)$-dp or
-$(O(r \epsilon), r \delta)$-dp.
-
-Let $\epsilon < 1$. We see that if the mechanism $N$ is
-$(\epsilon, \delta)$-dp on $Z^n$, then $M$ is
-$(2 r \epsilon, r \delta)$-dp, and if we run it over $k / r$
-minibatches, by Advanced Adaptive Composition theorem, we have
-$(\sqrt{2 k r \log \beta^{-1}} \epsilon + 2 k r \epsilon^2, k \delta + \beta)$-dp.
-
-This is better than the privacy guarantee without subsampling, where we
-run over $k$ iterations and obtain
-$(\sqrt{2 k \log \beta^{-1}} \epsilon + 2 k \epsilon^2, k \delta + \beta)$-dp.
-So with subsampling we gain an extra $\sqrt r$ in the $\epsilon$-part of
-the privacy guarantee. But, smaller subsampling rate means smaller
-minibatch size, which would result in bigger variance, so there is a
-trade-off here.
-
-Finally we define the differentially private stochastic gradient descent
-(DP-SGD) with the Gaussian mechanism
-(Abadi-Chu-Goodfellow-McMahan-Mironov-Talwar-Zhang 2016), which is (7)
-with the noise specialised to Gaussian and an added clipping operation
-to bound to sensitivity of the query to a chosen $C$:
-
-$$\theta_{t} = \theta_{t - 1} - \alpha \left(n^{-1} \sum_{i \in \gamma} \nabla_\theta \ell(x_i; \theta) |_{\theta = \theta_{t - 1}}\right)_{\text{Clipped at }C / 2} + N(0, \sigma^2 C^2 I),$$
-
-where
-
-$$y_{\text{Clipped at } \alpha} := y / (1 \vee {\|y\|_2 \over \alpha})$$
-
-is $y$ clipped to have norm at most $\alpha$.
-
-Note that the clipping in DP-SGD is much stronger than making the query
-have sensitivity $C$. It makes the difference between the query results
-of two *arbitrary* inputs bounded by $C$, rather than *neighbouring*
-inputs.
-
-In [Part 2 of this post](/posts/2019-03-14-great-but-manageable-expectations.html) we will use the tools developed above to discuss the privacy
-guarantee for DP-SGD, among other things.
-
-References
-----------
-
-- Abadi, Martín, Andy Chu, Ian Goodfellow, H. Brendan McMahan, Ilya
- Mironov, Kunal Talwar, and Li Zhang. "Deep Learning with
- Differential Privacy." Proceedings of the 2016 ACM SIGSAC Conference
- on Computer and Communications Security - CCS'16, 2016, 308--18.
- <https://doi.org/10.1145/2976749.2978318>.
-- Dwork, Cynthia, and Aaron Roth. "The Algorithmic Foundations of
- Differential Privacy." Foundations and Trends® in Theoretical
- Computer Science 9, no. 3--4 (2013): 211--407.
- <https://doi.org/10.1561/0400000042>.
-- Dwork, Cynthia, Guy N. Rothblum, and Salil Vadhan. "Boosting and
- Differential Privacy." In 2010 IEEE 51st Annual Symposium on
- Foundations of Computer Science, 51--60. Las Vegas, NV, USA:
- IEEE, 2010. <https://doi.org/10.1109/FOCS.2010.12>.
-- Shiva Prasad Kasiviswanathan, Homin K. Lee, Kobbi Nissim, Sofya
- Raskhodnikova, and Adam Smith. "What Can We Learn Privately?" In
- 46th Annual IEEE Symposium on Foundations of Computer Science
- (FOCS'05). Pittsburgh, PA, USA: IEEE, 2005.
- <https://doi.org/10.1109/SFCS.2005.1>.
-- Murtagh, Jack, and Salil Vadhan. "The Complexity of Computing the
- Optimal Composition of Differential Privacy." In Theory of
- Cryptography, edited by Eyal Kushilevitz and Tal Malkin,
- 9562:157--75. Berlin, Heidelberg: Springer Berlin Heidelberg, 2016.
- <https://doi.org/10.1007/978-3-662-49096-9_7>.
-- Ullman, Jonathan. "Solution to CS7880 Homework 1.", 2017.
- <http://www.ccs.neu.edu/home/jullman/cs7880s17/HW1sol.pdf>
-- Vadhan, Salil. "The Complexity of Differential Privacy." In
- Tutorials on the Foundations of Cryptography, edited by Yehuda
- Lindell, 347--450. Cham: Springer International Publishing, 2017.
- <https://doi.org/10.1007/978-3-319-57048-8_7>.